Đến nội dung

Nguyen Minh Hai

Nguyen Minh Hai

Đăng ký: 21-11-2013
Offline Đăng nhập: 28-07-2019 - 18:01
***--

#639955 VMF's Marathon Bất Đẳng Thức Olympic

Gửi bởi Nguyen Minh Hai trong 13-06-2016 - 00:20

 

Bài toán 31 (Võ Quốc Bá Cẩn). Cho các số thực dương $a,b,c$ thỏa mãn $a+b+c=5$. Chứng minh
$$a^2b+c^2a+2abc \leq 20.$$

 

Lời giải bài 31. Đặt $f(a,b,c)=a^2b+c^2a+2abc$

Ta xét $2$ trường hợp.

Nếu $a \geqslant c$, ta sẽ chứng minh 

$$f(a,b,c) \leqslant f(a,b,a)$$

$$\Leftrightarrow (a-c)(a+c+2b) \geqslant 0$$

 

Nếu $a \leqslant c$, ta sẽ chứng minh 

$$f(a,b,c) \leqslant f(c,b,c)$$

$$\Leftrightarrow b(c^2-a^2)+c^2(c-a)+2bc(c-a) \geqslant 0$$

 

Do đó ta chỉ cần chứng minh BĐT trong trường hợp $a=c=\frac{5-b}{2}$

$$\Leftrightarrow \left ( \frac{5-b}{2}\right )^2b+\left ( \frac{5-b}{2} \right)^3+2b\left (\frac{5-b}{2} \right )^2 \leqslant 20$$

$$\Leftrightarrow  b^3-9b^2+15b-7 \leqslant 0$$

$$\Leftrightarrow (b-7)(b-1)^{2} \leqslant 0$$

BĐT cuối luôn đúng do $b<5$

Bài toán được chứng minh. Xảy ra đẳng thức khi $a=c=2,b=1$

 

Bài toán 32.  (Vasile Cirtoaje) Cho các số thực dương $a,b,c,d$. Chứng minh rằng 

$$2(a^3+1)(b^3+1)(c^3+1)(d^3+1) \geqslant (1+abcd)(1+a^2)(1+b^2)(1+c^2)(1+d^2)$$




#633449 Inequalities From 2016 Mathematical Olympiads

Gửi bởi Nguyen Minh Hai trong 16-05-2016 - 14:38

 

Mình tổng hợp lại một số bài chưa có lời giải, mọi người cùng suy nghĩ thử nhé :)

 

Bài 28 (Russia). Cho bốn số thực dương $a, b, c, d$ thỏa mãn điều kiện $a+b+c+d=3.$ Chứng minh rằng $$\frac{1}{a^2}+\frac{1}{b^2}+\frac{1}{c^2}+\frac{1}{d^2}\le\frac{1}{a^2b^2c^2d^2},$$


$$\frac{1}{a^3}+\frac{1}{b^3}+\frac{1}{c^3}+\frac{1}{d^3}\le\frac{1}{a^3b^3c^3d^3}.$$

 

 

Lời giải 

 $(a)$       Giả sử $a \geqslant b \geqslant c \geqslant d$, BĐT cần chứng minh tương đương với

$a^{2}b^{2}c^{2}+a^{2}b^{2}d^{2}+a^{2}c^{2}d^{2}+b^{2}c^{2}d^{2} \leqslant 1$

Ta đưa BĐT về dạng đồng bậc

$a^{2}b^{2}c^{2}+a^{2}b^{2}d^{2}+a^{2}c^{2}d^{2}+b^{2}c^{2}d^{2} \leqslant \frac{1}{3^6}(a+b+c)^6$

Do $a \geqslant b \geqslant c \geqslant d$ nên ta

\[\begin{aligned} a^{2}b^{2}c^{2}+a^{2}b^{2}d^{2}+a^{2}c^{2}d^{2}+b^{2}c^{2}d^{2} &\leqslant a^{2}b^{2}c^{2}+a^{2}b^{2}d^{2}+a^{2}b^{2}cd+a^{2}b^{2}cd \\ &= a^{2}b^{2}(c+d)^{2} \\ &\leqslant \frac{1}{3^{6}}(a+b+c+d)^{6} \\ &=1\end{aligned}\]

Bài toán được chứng minh. $\square$ 

 $(b)$       Giả sử $a \geqslant b \geqslant c \geqslant d$, BĐT cần chứng minh tương đương với

$a^{3}b^{3}c^{3}+a^{3}b^{3}d^{3}+a^{3}c^{3}d^{3}+b^{3}c^{3}d^{3} \leqslant 1$

Ta đưa BĐT về dạng đồng bậc

$a^{3}b^{3}c^{3}+a^{3}b^{3}d^{3}+a^{3}c^{3}d^{3}+b^{3}c^{3}d^{3} \leqslant \frac{1}{3^9}(a+b+c)^9$

Theo BĐT $AM - GM$ ta

$ \frac{1}{3^9}(a+b+c+d)^9 \geqslant \frac{1}{3^9}\left(3\sqrt[3]{ab(c+d)} \right)^9 = a^3b^3(c+d)^3 $

Do đó ta chỉ cần chứng minh $a^{3}b^{3}c^{3}+a^{3}b^{3}d^{3}+a^{3}c^{3}d^{3}+b^{3}c^{3}d^{3} \leqslant a^3b^3(c+d)^3$ $\Leftrightarrow c^3d^3(a^3+b^3) \leqslant 3a^3b^3cd(c+d)$ $\Leftrightarrow c^2d^2(a^3+b^3) \leqslant 3a^3b^3(c+d)$

BĐT này luôn đúng do $a \geqslant b \geqslant c \geqslant d$

Bài toán được chứng minh. $\square$




#633413 Inequalities From 2016 Mathematical Olympiads

Gửi bởi Nguyen Minh Hai trong 16-05-2016 - 01:25

 

Mình tổng hợp lại một số bài chưa có lời giải, mọi người cùng suy nghĩ thử nhé :)

 

 

Bài 27 (Iran Second Round). Cho ba số thực $0<a \leqslant b \leqslant c.$ Chứng minh rằng
$$\frac{(c-a)^2}{6c}\leq \frac{a+b+c}{3}-\frac{3}{\frac{1}{a}+\frac{1}{b}+\frac{1}{c}}.$$

 

Hình gửi kèm

  • iran second round.jpg



#633411 Inequalities From 2016 Mathematical Olympiads

Gửi bởi Nguyen Minh Hai trong 16-05-2016 - 00:44

Một cách tiếp cận khác cho Bài 30

Bài 30 (EMMO). Cho ba số thực $a,b,c$ thỏa mãn điều kiện $a^2+b^2+c^2=4+abc.$ Chứng minh rằng $$(a^2+2)(b^2+2)(c^2+2)\ge 9+6(ab+bc+ca).$$
 

Đặt $t = \sqrt[3]{abc}$ thì từ giả thiết bài toán ta

$4+abc = a^{2}+b^{2}+c^{2} \geqslant 3\sqrt[3]{a^{2}b^{2}c^{2}}$

$\Leftrightarrow 4+t^3 \geqslant 3t^2  \Leftrightarrow  (t+1)(t+2)^2 \geqslant 0$

 

Từ đó suy ra $abc \geqslant -1$, kết hợp với giả thiết suy ra $a^{2}+b^{2}+c^{2} \geqslant 3$.

 

Ta BĐT quen thuộc sau $(a^{2}+2)(b^{2}+2)(c^{2}+2) \geqslant 3(a+b+c)^{2}   (1)$

 

Thật vậy, theo BĐT $Cauchy - Schwarz$ thì ta

 

$$(a+b+c)^{2} \leqslant (a^{2}+2)\left[1+\frac{(b+c)^2}{2}\right]$$

 

Do đó để chứng minh $(1)$ ta chỉ cần chứng minh

$$(b^{2}+2)(c^{2}+2) \geqslant 3\left[1+\frac{(b+c)^2}{2}\right]$$

 

$$\Leftrightarrow \frac{1}{2}(b-c)^2+(bc-1)^2 \geqslant 0$$

 

Do đó $(1)$ được chứng minh.

 

 Áp dụng $(1)$ thì ta quy bài toán về chứng minh

 

$$(a+b+c)^{2} \geqslant 3+2(ab+bc+ca)$$

 

$$\Leftrightarrow a^{2}+b^{2}+c^{2} \geqslant 3$$ 

 

BĐT này luôn đúng. Do đó bài toán được chứng minh 

 Xảy ra đẳng thức khi $a=b=c=-1$. $\square$




#633352 Inequalities From 2016 Mathematical Olympiads

Gửi bởi Nguyen Minh Hai trong 15-05-2016 - 21:23

 

Mình tổng hợp lại một số bài chưa có lời giải, mọi người cùng suy nghĩ thử nhé :)

 

 

Bài 23 (Romania JBMO TST 2016). Với $m,n$ là hai số tự nhiên và ba số thực $x,y,z$ thuộc $[0,1].$ Chứng minh rằng

\[0 \leqslant x^{m+n}+y^{m+n}+z^{m+n}-x^my^n-y^mz^n-z^mx^n \leqslant 1.\]
Đẳng thức xảy ra khi nào ?

 

$(a)$      Chứng minh $ x^{m+n}+y^{m+n}+z^{m+n} \geqslant x^my^n+y^mz^n+z^mx^n$

Gọi $(a,b,c)$ hoán vị của $(x,y,z)$ sao cho $a \geqslant b \geqslant c$

 

Khi đó ta $a^m \geqslant b^m \geqslant c^m$ $a^n \geqslant b^n \geqslant c^n$.

 

Do đó theo BĐT Hoán vị ta

$$a^ma^n+b^mb^n+c^mc^n \geqslant a^mb^n+b^mc^n+c^ma^n$$

$$a^ma^n+b^mb^n+c^mc^n \geqslant a^mc^n+b^ma^n+c^ma^n$$

ta

$$a^ma^n+b^mb^n+c^mc^n=x^{m+n}+y^{m+n}+z^{m+n}$$

$$x^my^n+y^mz^n+z^mx^n \in \lbrace a^mb^n+b^mc^n+c^ma^n , a^mc^n+b^ma^n+c^ma^n \rbrace$$

Do đó vế trái của BĐT được chứng minh. Xảy ra đẳng thức khi $x=y=z$

$(b)$         Chứng minh

$$x^my^n+y^mz^n+z^mx^n+1 \geqslant x^{m+n}+y^{m+n}+z^{m+n}$$

 

Giả sử $x=max\lbrace x,y,z \rbrace$ thì BĐT được viết lại thành

 

$$(1-x^{m+n})+y^n(x^m-y^m)+z^m(x^n-z^n)+y^mz^n \geqslant 0$$

 

BĐT cuối luôn đúng do $x=max\lbrace x,y,z \rbrace$ $x,y,z \in [0;1]$.

 

Vậy vế phải BĐT được chứng minh. Xảy ra đẳng thức khi $x=1,y=z=0$ các hoán vị. $\square$




#631722 ĐỀ THI OLYMPIC CHUYÊN KHOA HỌC TỰ NHIÊN 2016

Gửi bởi Nguyen Minh Hai trong 07-05-2016 - 12:05

 

ĐỀ THI OLYMPIC CHUYÊN KHOA HỌC TỰ NHIÊN 2016

Ngày 1 (07/05/2016)
Câu 1. Giải hệ phương trình $$\begin{cases}x + y + xy = 3 \\ y^{3} + 13y = 6x^{2} + 8\end{cases}$$
Câu 2. Tìm tất cả các bộ số nguyên dương $(x, y, z)$ thỏa mãn phương trình $$7^{x} + 3^{y} = 2^{z}$$
Câu 3. Cho tam giác $ABC$ nhọn có tâm nội tiếp $I$. Đường thẳng qua $I$ vuông góc với $AI$ cắt cạnh $CA, AB$ lần lượt tại $M, N$. Gọi $E$ đối xứng $C$ qua $M$, $F$ đối xứng $B$ qua $N$. Giả sử $E, F$ đều lần lượt thuộc các đoạn thẳng $CA, AB$. Gọi $(K), (L)$ lần lượt là đường tròn ngoại tiếp các tam giác $ICE, IBF$.

  • Chứng minh rằng $(K)$ và $(L)$ tiếp xúc nhau tại $I$.
  • Gọi $EF$ theo thứ tự cắt $(K), (L)$ tại $P, Q$ khác $E, F$. Chứng minh rằng $EP = FQ$.

Câu 4. Cho dãy số $(a_{n})_{n\in\mathbb{Z}^{+}}$ xác định như sau $$\begin{cases}a_{1} = 0 \\ a_{2} = 1 \\ a_{2n} = 2a_{n} + 1 \\ a_{2n + 1} = 2a_{n}\end{cases}$$ với mọi $n \in \mathbb{Z}^{+}$. Chứng minh rằng tồn tại vô số số nguyên dương $k$ sao cho $a_{k} = 2016$ và tìm số nguyên dương $k$ nhỏ nhất thỏa mãn điều này.






Nguồn

 

 

Tóm tắt 

Câu 1 :

Phương trình $(2)$ tương đương với 

$6(x-1)(x+1)=(y-1)(y^2+y+14)$

Rút $y=\frac{3-x}{x+1}$ từ $(1)$ thay vào phương trình trên ta được 

$$(\frac{2(x-1)}{x+1}((x+1)^{2}+(\frac{3-x}{x+1})^2+\frac{3-x}{x+1}+14)=0$$

 

Câu 2 : 

Dễ chứng minh được $z$ chẵn, $x,y$ khác tính chẵn lẽ

Xét 2 trường hợp $x$ chẵn , $y$ lẻ và $x$ lẻ, $y$ chẵn

Cả 2 trường hợp đều đưa về dạng 

$7^y=(2^{z'}-3^{x'})(2^{z'}+3^{x'})$ ... 

Bộ $(x,y,z)$ thỏa mãn là $(1;2;4)$

 

Câu 4 : Bằng quy nạp dễ dàng chứng minh được

 - $a_{2^n}=2^n-1$ 

 - $a_{2^n+1}=2^{n}-2 $

...

- $a_{2^n+k}=2^{n}-k-1$ với $k$ là số nguyên dương sao cho $2^n+k<2^{n+1}$

...

Từ đó dễ dàng suy ra kết quả bài toán.




#623931 $\sum_{x \in X}x =n^k$

Gửi bởi Nguyen Minh Hai trong 31-03-2016 - 21:59

Chứng minh rằng tồn tại tập hợp $A$ có $2016$ phần từ là các số nguyên dương sao cho với mọi tập $X\subset A, X \neq \varnothing$ thì ta có $\sum_{x \in X} x$ là lũy thừa với số mũ nguyên lớn hơn $1$ của một số nguyên dương nào đó.




#610657 ính tổng tất cả các số "tốt" không vượt quá 2016

Gửi bởi Nguyen Minh Hai trong 24-01-2016 - 00:46

Ta gọi một số nguyên dương là "tốt" nếu tổng các ước số của số đó là số chính phương. Tính tổng tất cả các số "tốt" không vượt quá 2016

 




#609809 $\frac{a^{2}+b^{2}+c^{2}}...

Gửi bởi Nguyen Minh Hai trong 19-01-2016 - 15:11

Cho a,b,c dương. Chứng minh

$\frac{a^{2}+b^{2}+c^{2}}{ab+bc+ca}+\frac{1}{3}\geq \frac{8}{9}\sum \frac{a}{b+c}$

BĐT $\Leftrightarrow \frac{a^2+b^2+c^2}{ab+bc+ca}-1 \geqslant \frac{8}{9}\left ( \sum \frac{a}{b+c}-\frac{3}{2} \right )$

 

        $\Leftrightarrow \sum \frac{(a-b)^2}{2(ab+bc+ca)} \geqslant \frac{8}{9}\sum \frac{(a-b)^2}{2(a+c)(b+c)}$

 

        $\Leftrightarrow (a-b)^2S_c+(b-c)^2S_a+(c-a)^2S_b \geqslant 0$

 

trong đó $S_c=\frac{1}{2(ab+bc+ca)}-\frac{4}{9(a+c)(b+c)}, S_b, S_a$ xác định tương tự và dễ dàng chứng minh được $S_a,S_b,S_c \geqslant 0$




#608877 Chứng minh rằng với mọi số nguyên dương $n$ đều tồn tại một hình vô...

Gửi bởi Nguyen Minh Hai trong 13-01-2016 - 23:21

Ta định nghĩa hình vuông "tốt" là một hình vuông có $4$ đỉnh là các điểm nguyên, đồng thời các đoạn thẳng nối tâm $O$ với tất cả các điểm nguyên trên biên và trong hình vuông chứa ít nhất một điểm nguyên khác hai đầu mút. Chứng minh rằng với mọi số nguyên dương $n$ đều tồn tại một hình vông tốt dạng $n.n$




#608215 Chứng minh rằng dãy $(p_n)$ bị chặn trên.

Gửi bởi Nguyen Minh Hai trong 09-01-2016 - 21:06

Cho dãy $(p_n)$ là dãy các số nguyên tố thỏa mãn với mọi $n \geqslant 3$ thì ta có $p_n$ là ước nguyên tố lớn nhất của $p_{n-1}+p_{n-2}+2000.$.

Chứng minh rằng dãy $(p_n)$ bị chặn trên. 




#600294 ĐỀ KIỂM TRA TOÁN 10 TRƯỜNG THPT CHUYÊN QUỐC HỌC 2015 - 2016

Gửi bởi Nguyen Minh Hai trong 27-11-2015 - 17:03

TRƯỜNG THPT CHUYÊN QUỐC HỌC - HUẾ                                         ĐỀ KIỂM TRA TOÁN 10

                            TỔ TOÁN                                                                                          Thời gian : 120 phút.

 

Câu 1: (4điểm) Cho $a,b,c$ là các số thực dương thỏa mãn $a+b+c=1$. Chứng minh:

                      $\frac{a^2+b^2}{c}+\frac{b^2+c^2}{a}+\frac{c^2+a^2}{b} \geqslant 2$

 

Câu 2: (6đ) Cho tam giác $ABC$, điểm $X$ thay đổi trên đường thẳng $BC$ sao cho $C$ nằm giữa $B$ và $X$. Gọi $M$ là trung điểm của $AB$. Đường tròn nội tiếp tam giác $ABX$ tiếp xúc với $BX, AX$ lần lượt tại $D,E$.

     a) Gọi $I$ là giao điểm của $DE$ và phân giác trong của góc $B$. Chứng minh $MI || BC$

     b) Các đường tròn nội tiếp các tam giác $ABX$ và $ACX$ cắt nhau tại $P$ và $Q$. Chứng minh rằng đường thẳng $PQ$ đi qua một điểm cố định.

 

Câu 3: (5đ) Phân chia tập $A=\left \{ 1;2;3;...;2016 \right \}$ thành $1008$ tập con rời nhau, mỗi tập gồm hai phần tử, ta gọi kích thước của mỗi tập con là tổng hai phần tử của nó. Gọi $x$ là số các tập con mà kích thước của chúng phân biệt từng đôi một và không vượt quá $2016$, gọi $S$ là tổng tất cả các phần tử của các tập con đó.

     a) Chứng minh rằng      $x(2x+1) \leqslant S \leqslant \frac{-x^2+4033x}{2}$

     b) Tìm giá trị lớn nhất của $x$.

 

Câu 4: (5đ) Với một số nguyên dương $n$ cho trước, gọi $f(n)$ là số nguyên dương nhỏ nhất sao cho $\sum_{k=1}^{f(n)} k$  là bội của $n$. Chứng minh rằng $f(n)=2n-1$ khi chỉ khi $n$ là lũy thừa của $2$

 

Spoiler




#599266 Chứng minh $x^{2}+y^{2}+z^{2}\leq x^...

Gửi bởi Nguyen Minh Hai trong 20-11-2015 - 19:43

Cho $x,y,z$ thuộc đoạn [0;1]. Chứng minh $x^{2}+y^{2}+z^{2} \leq x^{2}y+y^{2}z+z^{2}x$

Em nghĩ BĐT đúng phải là $x^2+y^2+z^2 \leqslant x^2y+y^2z+z^2x+1$

Ta có $x,y,z \in [0;1]$ do đó: 

$(x-1)(y-1)(z-1) \leqslant 0$   $\Leftrightarrow xyz-(xy+yz+zx)+(x+y+z)-1 \leqslant 0$

                $\Rightarrow xy+yz+zx \geqslant x+y+z+xyz-1 \geqslant x+y+z-1$

Lại có :   $x(x-1)(y-1) \geqslant 0$   $\Leftrightarrow x^2y \geqslant x^2+xy-x$

Xây dựng các BĐT tương tự rồi cộng theo vế ta được : 

$x^2y+y^2z+z^2x \geqslant x^2+y^2+z^2+(xy+yz+zx)-(x+y+z) \geqslant x^2+y^2+z^2-1 $ 

Do đó ta có đpcm.




#599265 Chứng minh $x^{2}+y^{2}+z^{2}+xyz\geq...

Gửi bởi Nguyen Minh Hai trong 20-11-2015 - 19:33

Cho $x,y,z \geq 0$ và $x+y+z=3$. Chứng minh $x^{2}+y^{2}+z^{2}+xyz\geq 4$

Đặt $\left\{\begin{matrix} p=x+y+z=3 & & & \\ q=xy+yz+zx & & & \\ r=xyz \leqslant 1 & & & \end{matrix}\right.$

Ta cần chứng minh :   $p^2-2q+r \geqslant 4$    $\Leftrightarrow 2q-r \leqslant 5$

Theo BĐT Schur ta có : $p^3-4pq+9r \geqslant 0$   $\Rightarrow q \leqslant \frac{27+9r}{12}$

Do đó:  $2q-r \leqslant \frac{27+9r}{6}-r = \frac{27}{6}+\frac{r}{2} \leqslant \frac{27}{6}+\frac{1}{2}=5$   




#599263 Chứng minh $x^{2}y+y^{2}z+z^{2}\leq...

Gửi bởi Nguyen Minh Hai trong 20-11-2015 - 19:28

Cho $x,y,z \geq 0$ và $x+y+z=1$. Chứng minh $x^{2}y+y^{2}z+z^{2}x\leq\frac{4}{27}$

Gọi $(a,b,c)$ là hoán vị của $(x,y,z)$ sao cho $a \geqslant b \geqslant c$.

Khi đó theo BĐT Hoán vị ta có : 

$x^2y+y^2z+z^2x=x.xy+y.yz+z.zx \leqslant a.ab+b.ac+c.bc$

                          $ = b(a^2+ac+c^2)$

                          $\leqslant \frac{1}{2}.2b(a+c)^2$

                          $\leqslant \frac{1}{2}.\left ( \frac{2a+2b+2c}{3} \right )^3 = \frac{4}{27}$

Xảy ra đẳng thức khi $a=b=c=\frac{1}{3}$

[/spoiler] Em Hải 10 Toán 1 đây thầy ! Phải thầy Nghĩa ko ạ ? :3 [spoiler]